help me please, i don't know this

Answers

Answer 1

its full form is portable document format


Related Questions

One angle of a triangle is equal to the sum of the remaining angles. If the ratio of measures of the ren
is 2:1, find the measures of the three angles of the triangle.

Answers

9514 1404 393

Answer:

  90°, 60°, 30°

Step-by-step explanation:

The remaining angles have a ratio of 2:1, so total 3 "ratio units". The first angle is equal to that sum: 3 ratio units, so all of the angles together total 3+2+1 = 6 ratio units. The total of angles is 180°, so each ratio unit is 180°/6 = 30°.

The first angle is 3 ratio units, or 90°.

The second angle is 2 ratio units, or 60°.

The third angle is half that, or 30°.

The three angles are 90°, 60°, 30°.

What is the complete factorization of the polynomial below?
x3 + 4x2 + 16x + 64

Answers

Answer: (x+4) ( x-4i)(x+4i)

Discussion:

Factor the polynomial:

x^3+4x^2+16x+64 =  (x +4 ) ( x^2 + 16)     (*)

Factor x^2 +1 6 over the complex numbers:

x^2 + 16 = (x -4i)(x+4i)                               (**)

Combing (*) and (**) gives the full factorization

(x+4) ( x-4i)(x+4i)

plz help me with this question

What is one important role that ocean waters have in heating Earth?

absorb and transport solar energy
transport nutrients and waste
create and emit solar energy
wear away rock and soil






also support me on on ubute SadSpade4679

Answers

Answer:

The ocean absorbs heat energy from the sun

Step-by-step explanation:

Yw :)

Answer:

The ocean absorbs heat energy from the sun.

Step-by-step explanation:

The salt content in snack bags of pretzels is Normally distributed, with a mean of 180 mg and a standard deviation of 15 mg. Eighty four percent of bags have a salt content higher than which value?


Find the z-table here.


165.2 mg

179.2 mg

187.0 mg

194.9 mg

I think its (A), 165.2mg

Answers

Answer: Yes you are correct. The answer is choice A

============================================================

Explanation:

If you used the z-table, you should find that P(Z < 1) = 0.84 approximately.

So by symmetry, P(Z > -1) = 0.84 approximately as well.

We'll convert the z score z = -1 into its corresponding x score

z = (x-mu)/sigma

-1 = (x-180)/15

-15 = x-180

x-180 = -15

x = -15+180

x = 165

We don't land on any of the answer choices listed, but we get fairly close to 165.2, which is choice A. So you are correct.

I have a feeling that the table you have is probably more accurate than the one I'm using, so it's possible that you'd land exactly on 165.2 when following the steps above.

Answer:

194.9

Step-by-step explanation:

ON EDG

04.10 Which statement about the graph is true? CO 7 E 5 4 3 2 1 2 3 4 5 6 7 8 x The graph shows a proportional relationship because it is a line, and the difference between each point is the same. The graph shows a proportional relationship because it is a line, and each x-value is a multiple of 2. The graph does not show a proportional relationship because each point written as a ratio gives a different value. O The graph does not show a proportional relationship because a line that increases by 1 in the y-value cannot have a constant of proportionality.​

Answers

Answer:

c

Step-by-step explanation:

cheese is rotten milk

What is the common difference for this arithmetic sequence?
-6,-1,4,9,14,...
A. 6
B. 4
C. 5
D. 3
SUBMIT

Answers

Answer:

5 is the answer to your question

Step-by-step explanation:

the numbers are increasing by +5

what is the mean of this graph?

Answers

Answer:

Step-by-step explanation:

Mean: sum of terms/ number of terms

4 + 2 + 3 + 1 + 1 + 5  = 16

Numbr of terms = 6

16/6 = 2.66

Answered by g a u t h m a th

Answer:

2 2/3

Step-by-step explanation:

The mean is the average of the values

The values on the graph are 4, 2, 3, 1, 1, and 5

Average is sum/number of values

4+2+3+1+1+5=16

16/6=2 2/3

-.p+p⎯.+p Simplify, please.

Answers

Answer:

34.5p-2.75

Step-by-step explanation:

First add -0.5p and 12p together which is 11.5p, then add 23p with 11.5p which is 34.5p And -2.75 remains the same

So the answer is 34.5p-2.75

Answer:

34.5p-2.75

Step-by-step explanation:

-0.5p+12p-2.75+23p=34.5p-2.75

One year, Alex bought an antique car for his birthday. During the first year he owned it, the
value of the car gained 10%. During the second year, the value of the car gained another
15% from the previous year. If the value of the car is now $37,950.00, how much did Alex
originally pay for his car?

Answers

Answer:

28462.5

Step-by-step explanation:

During the first year it gained 10% and during his second year he gained 15% so you first add those and you get 25%.

Then you multiply 25% with 37,950.

25/100 * 37950 = 948,750/100

= 9487.5

To get the original amount you subtract 9487.5 from 37,950

37,950 - 9487.5 = 28462.5

So the original amount was 28462.5

Find the length of AC again

Answers

Answer:

B

Step-by-step explanation:

side Ac is the opposite of angle B, therefore use the sin ratio to find ac since the hypotenuse is also given

sin B=opposite/hypotenuse

sin 27=ac/15

ac=sin27×15

=6.81

I hope this helps

Answer:

AC = 6.81

Step-by-step explanation:

∠BAC = 90° - 27° = 63°

Cos 63° =[tex]\frac{AC}{AB}[/tex] = 0.454

AC = cos 63° * AB = 0,454 * 15 = 6.81

Solving Equations by Dividing 2) 9x= -135 Solve for x. 0 -144 O 126 O 15 0 -15

Answers

X = 15
Explanation:
9x = -135
Divide by 9 on both sides
X = -15
It is -15 since 135 is negative

[tex]\huge\text{Hey there!}[/tex]

[tex]\huge\boxed{\mathsf{9x = -135}}[/tex]

[tex]\huge\boxed{\text{DIVIDE 9 to BOTH SIDES}}[/tex]

[tex]\huge\boxed{\mathsf{\dfrac{9x}{9}= \dfrac{-135}{9}}}[/tex]

[tex]\huge\boxed{\mathsf{\bullet \ CANCEL: \dfrac{9}{9}\ because\ it \ gives\ you\ 1}}[/tex]

[tex]\huge\boxed{\bullet\ \mathsf{KEEP: \dfrac{-135}{9}\ because\ it\ helps\ solve \ for}}\\\huge\boxed{\mathsf{the\ x-value}}[/tex]

[tex]\huge\boxed{\mathsf{x = \dfrac{-135}{9}}}\\\\\huge\boxed{\mathsf{\dfrac{-135}{9}= x}}}[/tex]

[tex]\huge\boxed{Simplify \ it\uparrow}[/tex]

[tex]\huge\boxed{\mathsf{x = \bf -15}}[/tex]

[tex]\huge\boxed{\textsf{Therefore, your answer is: Option D. -15 }}\huge\checkmark[/tex]

[tex]\huge\text{Good luck on your assignment \& enjoy your day!}[/tex]

~[tex]\huge\boxed{\frak{Amphitrite1040:)}}[/tex]

A small boat can travel at 28 per hour how many hours will it take to go across the bay that is 56 miles wide

Answers

Answer:

2 hours

Remember that time = distance/rate

The distance you need to cover is 56 miles, while you go 28 miles per hour. Using these, we get this:

time=56/28

time=2

So it will take two hours to go across a 56 mile wide bay at 28 mph.

Step-by-step explanation:

_+3=-9 pls help I will give branlyiest

Answers

Answer:

-12

Step-by-step explanation:

? + 3 = -9

Subtract 3 from -9

-9 - 3 = -12

? = -12

factorise completely 4x^(2 )(x + 1) - 6x (x+1)

Answers

Answer:

[tex] {4x}^{2} (x + 1) - 6x(x + 1) \\ = (x + 1)(4 {x}^{2} - 6 x ) \\ = (x + 1)(2x)(2x - 3)[/tex]

explanation:

first choose the common factor by observation, it is (x + 1):

factorise it out:

= (x + 1)(4x² - 6x)

by observation in (4x² - 6x), common factor is 2x.

Factorise 2x out:

= (x + 1)[2x(2x - 3)]

Answer:

(4x2-6x) (x+1)

now common factor is (x+1) ,so,(4x2-6x) (x+1)

A shipping container in the shape of a rectangular prism is 60 feet long, 45 feet wide, and 14 feet tall. What is the volume of the shipping container? *

Answers

V= 37800 ft^3

The formula for the volume of a rectangular prism is: width x height x length and is expressed in cubed units

A few more problems and then I’m done

Answers

Answer:

((c)).g(x) = 3 × 2^x +2..

A building is 1 ft from an 8-ft fence that
surrounds the property. A worker wants
to wash a window in the building 11 ft
from the ground. He plans to place a
ladder over the fence so it rests against
the building. (See the figure.) He
decides he should place the ladder 8 ft
from the fence for stability. To the
nearest tenth of a foot, how long a
ladder will he need?

Answers

Answer:

This question is mainly about pythagorean theorm as if we just analyse this question we see the information given to us are

The height of the building = 11 ft The distance in between the fence and the building = 1 ftThe distance of ladder from the fence = 8 ft

Now it is said to find out the length of ladder as because the ladder is resting against the buiding so it will generally form an acute angle with the ground and as we can take the ladder to be as the hypotenuse of a right angled triangle in which the length of the perpendicular building forms one of the leg and the distance of the building from the point where the ladder is kept is said to be another leg

Now we get the measure of the vertical leg = height of the buiding = 11 ft

and another leg = distance in between the fence and the point on which the ladder is kept + the distance in between the fence and building

= (8+1)ft =9ft

Now we know hypotenuse = √base^2+√height^2

= √9^2+√11^2

= √81+√121

= √ 202

= 14.21 ft

Therefore the length of the ladder = 14.21 ft

Hope it helps you

The length of the ladder should be 11.1 feet long.

Let's denote the length of the ladder as L.

According to the given information:

The distance from the building to the fence = 1 ft.

The height of the window = 11 ft.

The distance from the ladder to the fence = 8 ft.

Using the Pythagorean theorem, we can set up the equation:

[tex]L^2 = (1^2) + (11^2)[/tex]

Simplifying:

[tex]L^2 = 1 + 121[/tex]

[tex]L^2 = 122[/tex]

Taking the square root of both sides:

[tex]L = \sqrt{122}[/tex]

[tex]L = 11.05 ft[/tex]

Therefore, the worker will need a ladder approximately 11.1 feet long.

To learn more on trigonometry click:

https://brainly.com/question/25122835

#SPJ4

6. Find the missing side. Round to the nearest tenth ​

Answers

Answer:

x = 7.6

Step-by-step explanation:

We know the opposite side and the adj side and this is a right triangle

tan theta = opp / adj

tan 66 = 17/x

x tan 66 = 17

x = 17 /tan 66

x=7.56888

To the nearest tenth

x = 7.6

tanØ=Perpendicular/Base

tan66=17/xx=17/tan66x=7.57x=7.6

Yoda Soda is the intergalactic party drink that will have all your friends saying, "Mmmmmm, good this is!"
You are throwing a party, and you need 555 liters of Yoda Soda for every 12 guests.
If you have 363636 guests, how many liters of Yoda Soda do you need?

Answers

Answer:

16818165

Step-by-step explanation:

12 guests equal 555 liters

total guests 363636

divide 363636 by 12

result is 30303

multiply 30303 times 555

total is 16818165

After running 3/4 of a mile tess has only run 1/3 how long is the race in miles but I want to know how you did it

Answers

Take 3/4 and 1/3 multiply the bottom, thats your denominator. Then since you multiplied 4•3, multiply the top by three.

What is the area of this figure?

Answers

Answer:

22

Step-by-step explanation:

(5x2) + (3x2) + (3x2)

22 square units

Answer from Gauthmath

Smart phone: Among 239 cell phone owners aged 18-24 surveyed, 103 said their phone was an Android phone. Part: 0 / 30 of 3 Parts Complete Part 1 of 3 (a) Find a point estimate for the proportion of cell phone owners aged 18-24 who have an Android phone. Round the answer to at least three decimal places. The point estimate for the proportion of cell phone owners aged 18-24 who have an Android phone is .

Answers

Answer:

The point estimate for the proportion of cell phone owners aged 18-24 who have an Android phone is 0.4137.

Step-by-step explanation:

The point estimate is the sample proportion.

Sample proportion:

103 out of 249, so:

[tex]p = \frac{103}{249} = 0.4137[/tex]

The point estimate for the proportion of cell phone owners aged 18-24 who have an Android phone is 0.4137.

Find the values for x and y using the diagram below. Explain what geometric relationships you used to solve the problem.
You DO NOT have to write a proof. An example would be "linear pair" or "right angles= 90◦
".

Answers

Answer:

2x, a linear pair

Step-by-step explanation:

2/5x/3/4 = 7/4 = 1 3/4

The sum of a number x and
eleven

Answers

Answer:

what is the sum.

Step-by-step explanation:

Take the sum - 11 =x

Complete the function table.

Answers

Answer:

B

Step-by-step explanation:

The function given is f(n) = n-3. Plug in n=0 and you will get an output - 3. Plug in n=2 and you will get an output - 1. Hence table B is the answer .

[tex]\int\limits^9_3 {\frac{1}{(x+21)\sqrt{x+22} } } \, dx[/tex]

Answers

Let y = x + 22 and dy = dx, so the integral becomes

[tex]\displaystyle \int_3^9 \frac{\mathrm dx}{(x+21)\sqrt{x+22}} = \int_{25}^{31} \frac{\mathrm dy}{(y-1)\sqrt{y}}[/tex]

Now let z = √y, so that z ² = y. Then 2z dz = dy, and the integral becomes

[tex]\displaystyle \int_3^9 \frac{\mathrm dx}{(x+21)\sqrt{x+22}} = \int_{\sqrt{25}}^{\sqrt{31}} \frac{2z}{(z^2-1)z} \\\\ = \int_5^{\sqrt{31}} \frac{2}{z^2-1}\,\mathrm dz[/tex]

Expand the integrand into partial fractions:

[tex]\dfrac{2}{z^2-1} = \dfrac1{z-1}-\dfrac1{z+1}[/tex]

Then we have

[tex]\displaystyle \int_3^9 \frac{\mathrm dx}{(x+21)\sqrt{x+22}} = \int_5^{\sqrt{31}}\left(\frac1{z-1}-\frac1{z+1}\right)\,\mathrm dz \\\\ = \left(\ln|z-1|-\ln|z+1|\right)\bigg|_5^{\sqrt{31}} \\\\ =\left[\ln\left|\frac{z-1}{z+1}\right|\right]\bigg|_5^{\sqrt{31}} \\\\ =\ln\left(\frac{\sqrt{31}-1}{\sqrt{31}+1}\right) - \ln\left(\frac{4}{6}\right) \\\\ =\ln\left(32-2\sqrt{31}\right) - \ln\left(\frac23\right) \\\\ =\boxed{\ln\left(48-3\sqrt{31}\right)}[/tex]

I need help ASAP thank you

Answers

9514 1404 393

Answer:

  C

Step-by-step explanation:

The graph shows two vertical asymptotes, so the relevant function will be zero in the denominator for two different x-values. The only possibility is ...

  [tex]F(x)=\dfrac{1}{(x-1)(x+1)}[/tex]

The starting line up for a basketball team is to consist of two forwards and three guards. Two brothers are on the team. Matthew is a forward and Tony a guard. There are four forwards and six guards from which to choose the line up. If the starting players are chosen at random, what is the probability that the two brothers will end up in the starting line up

Answers

Answer:

0.25 = 25% probability that the two brothers will end up in the starting line up

Step-by-step explanation:

A probability is the number of desired outcomes divided by the number of total outcomes.

The order in which the players are chosen is not important, which means that the combinations formula is used to solve this question.

Combinations formula:

[tex]C_{n,x}[/tex] is the number of different combinations of x objects from a set of n elements, given by the following formula.

[tex]C_{n,x} = \frac{n!}{x!(n-x)!}[/tex]

Desired outcomes:

Matthew plus another forward from a set of 3.

Tony plus another two guards from a set of 5.

So

[tex]D = C_{3,1}C_{5,2} = \frac{3!}{1!2!} \times \frac{5!}{2!3!} = 3*10 = 30[/tex]

Total outcomes:

Two forwards from a set of 4.

Three guards from a set of 6.

So

[tex]T = C_{4,2}C_{6,3} = \frac{4!}{2!2!} \times \frac{6!}{3!3!} = 6*20 = 120[/tex]

What is the probability that the two brothers will end up in the starting line up?

[tex]p = \frac{D}{T} = \frac{30}{120} = 0.25[/tex]

0.25 = 25% probability that the two brothers will end up in the starting line up

[tex](4 - 3i)x^{2}[/tex]

Answers

Answer:

(4−3i)x^2

=4x^2 - 3ix^2

If you asking for simplifying:
4x square 2 - 3x square 2 I

help asap!!
Find the length of AB
A. 2.89
B. 33.13
C. 378.63
D. 377.19

Answers

Answer:

C

Step-by-step explanation:

[tex] \sin( 5 ^{o} ) = \frac{33}{ab} \\ ab = 378.63[/tex]

The answer is C……………….
Other Questions
What is the least common denominator that will allow you to combine the constant terms? 10 21 35 or 42 HELP PLEASEEE!!!! ASAP!!!!! 7. If the supplement of an angle is 69, then find its complement. Find the length of the arc round your answer to the nearest 10th What is figurative language?A. MetaphorB. Imaginative writingC. Flowery language which has realistic descriptionsD. Subtle language that cannot be taken literally a rice cooker was sold for $60 after a discount of 60% waht was the usual price of the rice cooker Which of the following is the number of sides for a regular polygon that willnot form a regular tessellation?A. 6B. 4C. 7D. 3 What is the second integral of [tex] \frac{1}{x} [/tex]? arrange the resistance r1 r2 and r3 in increasing order. A state of economic scarcity exists when consumers: A. do not have enough resources to satisfy all of their wants.B. save their money in banks instead of spending it. C. borrow too much money to buy unnecessary products. D. have their economic freedoms restricted by the government. Omg who can help me? Its due tomorrow Based on how they each acted to the trials,witch substance (s)would most likely be metallic Which area appears only on the posterior part of the body? a)scapular b)patellar c)umbilical d)sternal D. He no longer thinks about her. Can you do this question? A particular strain of bacteria triples in population every 45 minutes Assuming you start with 50 bacteria in a Petri dish how many bacteria will there be in 4.5 hours Noelle has containers that each hold one gallon of liquid. She used these containers to dispose of waste from the chemistry laboratory. She collected 3 quarts 1 pint of liquid from the first table, 4 quarts from the second table, 2 quarts from the third table , and 3 quarts 1 pint from the fourth table. What is the least number of containers Alyssa needed to collect the waste ? ( 4 quarts = 1 gallon , 2 pints = 1 quart ) Please help 50 points! Maricopa's Success scholarship fund receives a gift of $ 160000. The money is invested in stocks, bonds, and CDs. CDs pay 4.5 % interest, bonds pay 5.8 % interest, and stocks pay 8.4 % interest. Maricopa Success invests $ 15000 more in bonds than in CDs. If the annual income from the investments is $ 10450 , how much was invested in each account What is the most effective method interest groups have for achieving their legislative and judicial goals